Breast Reduction 01-22 Flashcards

1
Q

Based on a systematic evidence review, which of the following is the most appropriate preoperative recommendation for women of average breast cancer risk undergoing reduction mammaplasty?

A) Annual mammography screening for women 45 to 54 years of age
B) Biennial breast MRI screening for women over age 55 years
C) BRCA testing
D) Establish a baseline mammogram for women over age 30 years
E) Screening breast thermography

A

The correct response is Option A.

The American Cancer Society 2015 guidelines for women undergoing reduction mammaplasty based on evidence-based medicine include: 1) annual screening for women aged 45 to 54 years; 2) biennial screening for women 55 years of age and older; 3) women aged 40 to 44 years have the opportunity to begin annual screening; 4) clinical breast examination is not indicated for cancer detection and surveillance. BRCA testing is currently recommended for high-risk familial inheritance concerns. Breast thermography and MRI are not used for screening patients of average risk.

How well did you know this?
1
Not at all
2
3
4
5
Perfectly
2
Q

A 32-year-old woman presents with symptomatic macromastia. Family history includes premenopausal breast cancer in her aunt. The patient is scheduled for bilateral reduction mammaplasty (using an inferior pedicle with wise pattern skin incision), and consent is obtained. During the procedure, a 3 × 3-cm area of suspicious tissue on the left upper outer quadrant is noted. Frozen section study confirms a 5-mm ductal carcinoma with free margins. Which of the following is the most appropriate next step in management of this patient?

A) Completion of reduction mammaplasty bilaterally, along with immediate left axillary sentinel node study
B) Completion of reduction mammaplasty bilaterally, along with routine orientation of the specimens, marking the cancer site with sutures and surgical metal clips
C) Completion of reduction mammaplasty bilaterally, followed by scheduling the patient for immediate adjuvant chemoradiotherapy
D) Completion of reduction mammaplasty bilaterally with excision of 3-cm extra margins of the cancer site
E) Completion of reduction mammaplasty on the right side, followed by left-sided mastectomy

A

The correct response is Option B.

The incidence of occult breast cancer detected by reduction mammaplasty is 0.06 to 5.45%. Management of the occult breast cancer depends on the history of breast cancer, surgical margin, and family history. Preoperative thorough history, clinical examination, and counseling regarding the risk for and implications of finding occult breast cancer are necessary. All women aged 35 years and older with positive family history or personal history of breast cancer should have a screening mammogram before surgery; if there is no family history, women aged 40 years and older should have a screening mammogram before surgery. Routine orientation of the specimens for pathologic analysis and en bloc resection should be performed in patients aged 30 years and older and in any patients with a personal or family history of breast cancer. Although mastectomy is the most common procedure performed after an incidental finding of occult breast cancer during reduction mammaplasty, multidisciplinary evaluation and tests are necessary before performing any surgical procedures (mastectomy, sentinel node study) or any plan for chemotherapy, hormone therapy, or radiation therapy. Also, lack of consent for other surgical procedures would prohibit the surgeon from adding another procedure at the time of breast reconstruction. Excision of the 3-cm extra margins is not the standard of care for 5-mm invasive breast cancer.

How well did you know this?
1
Not at all
2
3
4
5
Perfectly
3
Q

A 15-year-old girl is referred to the office because of a 7-month history of asymptomatic, rapid enlargement of the right breast. Physical examination shows a large, palpable mass occupying the lower half of the right breast. There is marked nipple-areola complex stretching and prominent dilated veins. Photographs are shown. Mammogram and ultrasound show a dense, circumscribed, homogeneous 15-cm mass in the right breast. Which of the following is the most likely diagnosis?

A) Carcinoma
B) Cyst
C) Giant fibroadenoma
D) Juvenile breast hypertrophy
E) Phyllodes tumor

A

The correct response is Option C.

This patient has a fibroadenoma, the most common breast neoplasm in adolescent females. Giant fibroadenoma are typically solitary, firm, nontender, and present as a rapid asymmetric breast enlargement with prominent overlying veins and occasional pressure-induced skin ulceration. These lesions are larger than 5 cm and present at or soon after the onset of puberty. These lesions are typically treated with enucleation using breast reduction techniques for optimal symmetry with the contralateral breast. Mastectomy is not indicated, and no other adjuvant therapy is necessary. Smaller fibroadenomas may be watched conservatively, with minimal risk for malignant transformation. Surgical intervention is indicated in cases of mastodynia, neck/back pain secondary to large size, difficulty with clothing due to asymmetry, and to alleviate patient concern.

The differential diagnosis would include phyllodes tumor, breast hypertrophy, or cystic enlargement. Carcinoma would be unlikely in this age demographic. Phyllodes tumors are large, benign tumors that typically occur in the perimenopausal patient. They are histologically distinct from fibroadenomas, and transformation of a fibroadenoma to a phyllodes tumor is exceptionally rare.

Juvenile breast hypertrophy may present as unilateral or bilateral breast enlargement. The enlargement is diffuse without evidence of a discrete mass or nodularity. Juvenile breast hypertrophy typically presents in early puberty, rarely regresses spontaneously, and is much more severe than simple breast hypertrophy. The underlying cause is attributed to estrogen stimulation at the onset of the first menses. Treatment is reduction mammoplasty.

How well did you know this?
1
Not at all
2
3
4
5
Perfectly
4
Q

A healthy 18-year-old woman presents for evaluation for reduction mammaplasty. Physical examination of the breasts shows a palpable area of fibrocystic tissue in the left axilla. The patient states that this tissue has been present since menarche and often becomes swollen and slightly tender during menses. Which of the following is the most likely developmental origin of this tissue?

A) Failure of mammary ridge involution
B) Hypertrophy of lactiferous duct formation
C) Incomplete mesodermal resorption
D) Overdevelopment of apocrine and sebaceous glands

A

The correct response is Option A.

Accessory breast tissue, or polymastia, results from incomplete involution of the ectodermal ridge outside the main mammary buds. During embryologic development, the breast forms from paired ectodermal ridges. Fifteen to twenty of these buds initially appear but then undergo apoptosis, leaving a pair of main mammary buds at the level of the fourth to fifth intercostal space. These mammary buds will become breasts. However, accessory breast tissue, such as in this patient, develops when a portion of the ectodermal ridges fails to involute during the seventh week of gestation. The other choices do not accurately describe the developmental origin of accessory breast tissue.

How well did you know this?
1
Not at all
2
3
4
5
Perfectly
5
Q

A 35-year-old woman undergoes bilateral reduction mammaplasty for treatment of symptomatic macromastia. Medical history includes chronic neck and back pain for 15 years. She wears a size 36H brassiere. The pathology specimen shows ductal carcinoma in situ completely excised with 2-mm margins. Postoperative MRI shows no additional abnormalities. Which of the following is the most appropriate next step to adequately treat this patient’s breast cancer?

A) Chemotherapy
B) Formal lumpectomy for adequate margins
C) Radiation therapy
D) Sentinel lymph node dissection
E) No additional treatment necessary

A

The correct response is Option C.

An incidental finding of breast cancer on breast reduction specimen final pathology can lead to significant anxiety for both the patient and the plastic surgeon. To mitigate this anxiety and provide the best care and support for the patient, plastic surgeons should be aware of the current guidelines for treatment of breast cancer. In this patient, there are two viable options for adequate treatment similar to most breast cancers after the diagnosis is confirmed by biopsy with equivalent 5-year survival rates: lumpectomy and adjuvant radiation versus completion mastectomy. Ductal carcinoma in situ (DCIS) requires excision with margin greater than or equal to 2 mm for adequate extirpation. Because this patient has adequate margins with the breast reduction specimen, she would only require adjuvant radiation therapy unless she elects to have completion mastectomy with or without reconstruction. Chemotherapy is reserved for invasive tumors with potential systemic spread. Sentinel lymph node dissection is not required with in situ cancers that are noninvasive.

How well did you know this?
1
Not at all
2
3
4
5
Perfectly
6
Q

A 45-year-old woman presents for consultation for reduction mammaplasty. On examination, a 1-cm mass in the upper outer quadrant is noted. Which of the following is the most appropriate initial imaging test for this patient?

A) Diagnostic mammography
B) MRI
C) Positron emission tomography
D) Sestamibi scan
E) Thermography

A

The correct response is Option A.

Diagnostic mammography is the initial imaging modality of choice for a clinically detected palpable breast mass in a woman aged 40 years or older. For a woman under the age of 30 years, breast ultrasonography would be the first choice. For a woman aged 30 to 39 years, either modality would be acceptable for initial evaluation. It is essential that there be correlation between the imaging and area of palpable concern to confirm the correct finding is being evaluated. Regardless of palpable findings, any highly suspicious breast mass that is detected by imaging warrants biopsy. Further, any highly suspicious breast mass that is detected by palpation warrants biopsy. Thermography has not been approved for screening.

How well did you know this?
1
Not at all
2
3
4
5
Perfectly
7
Q

An otherwise healthy 29-year-old woman is scheduled to undergo bilateral reduction mammaplasty. Medical history includes symptomatic macromastia and significant breast hypertrophy. The operative plan includes a Wise pattern, inferior pedicle breast reduction, and infiltration of the incisions and parenchyma with a dilute epinephrine solution at a concentration of 1:500,000. The benefit of infiltration of the operative sites is to decrease which of the following?

A) Intraoperative blood loss
B) Operative time
C) Postoperative drainage
D) Risk of hematoma
E) Risk of infection

A

The correct response is Option A.

The use of epinephrine-containing wetting solutions has been used extensively in plastic surgery to minimize blood loss complications in many types of surgery, including breast reduction. Injection of a dilute epinephrine wetting agent can significantly reduce blood loss. No change in operative time, risk for infection, hematoma or postoperative bleeding, or drainage outputs has been shown.

How well did you know this?
1
Not at all
2
3
4
5
Perfectly
8
Q

A 52-year-old woman undergoes preoperative mammogram. Medical history includes symptomatic macromastia. Suspicious calcifications are identified, and a core-needle biopsy is performed. Which of the following results necessitates an excisional biopsy prior to proceeding with the reduction mammaplasty?

A) Atypical lobular hyperplasia
B) Fibroadenoma
C) Papilloma without atypia
D) Pseudoangiomatous stromal hyperplasia
E) Radial scar

A

The correct response is Option A.

National Comprehensive Cancer Network (NCCN) protocol recommends excisional biopsy following detection of the following high risk lesions: papillomas with atypia, atypical ductal hyperplasia (ADH), atypical lobular hyperplasia (ALH), and ductal carcinoma in situ (DCIS). These lesions, initially diagnosed on vacuum-assisted core-needle biopsy, progress to carcinoma of the breast on excisional biopsy in 10% to 39% of cases. Also, any proliferative lesion with atypia or any biopsy result that is discordant with its radiologic image should also undergo excisional biopsy. If a radial scar has been completely excised and definitively diagnosed, it does not require an excisional biopsy.

Once a lesion has been confirmed benign, it is safe to proceed with reduction mammaplasty. It is important to note that a woman with a history of ADH, ALH, or lobular carcinoma in situ (LCIS) will have an increased risk for developing in situ or invasive breast carcinoma over her lifetime. These patients should, at a minimum, undergo yearly mammographic screening as well as possible breast magnetic resonance imaging and risk reducing strategies including endocrine therapy.

Pseudoangiomatous stromal hyperplasia (PASH) is a benign breast lesion characterized histologically by dense collagenous stroma with spindle cell-lined spaces that appear like capillaries. Microscopic disease may be found incidentally, or it may be associated with a palpable mass. It has not been demonstrated to increase risk for subsequent breast carcinoma development.

How well did you know this?
1
Not at all
2
3
4
5
Perfectly
9
Q

Which of the following is the dominant blood supply to the nipple-areola complex in a superomedial pedicle breast reduction?

A) Anterior lateral perforators
B) Dermal plexus
C) Internal mammary perforators
D) Lateral thoracic system perforators

A

The correct response is Option C.

The vascular supply to the nipple areola complex in a superomedial breast reduction is perforators of the internal mammary artery.

Anterior lateral perforators are divided during superomedial pedicle technique.

Lateral pedicle (Skoog technique) is relied on lateral thoracic system perforators.

Dermal plexus could be perfuse by any of the above arteries but it finally depends on which artery has not been sacrificed during the creation of the pedicle. In superomedial pedicle technique, the lateral thoracic and anterior intercostals have been already sacrificed.

References

How well did you know this?
1
Not at all
2
3
4
5
Perfectly
10
Q

A 45-year-old woman with macromastia presents for breast reduction surgery. BMI is 34 kg/m2. Physical examination shows grade III ptosis with bilateral bra strap grooving. A photograph is shown. Breast reduction mammaplasty of approximately 1000 g per breast using a Wise pattern and inferior pedicle technique is planned. Which of the following factors is most likely to increase this patient’s risk for perioperative complications?

A) BMI
B) Degree of ptosis
C) Inferior pedicle
D) Resection volume
E) Wise pattern

A

The correct response is Option D.

According to the Breast Reduction Assessment: Value and Outcomes (BRAVO) study, the overall complication rate for breast reduction was 43% with delayed healing as the most common. This complication was correlated directly with average preoperative breast volume, average resection weight/breast, and smoking, and was correlated inversely with age. Analysis associated resection weight as the sole variable for increased risk of complications and with absolute number of complications (greater than 847 g). Each 10-fold increase in resection weight increased the risk of complication 4.8 times and increased the risk of delayed healing 11.6 times. Degree of ptosis was not correlated with delayed healing. Vertical incision techniques were associated with an increased complication frequency without a link to specific complications.

How well did you know this?
1
Not at all
2
3
4
5
Perfectly
11
Q

A previously healthy 13-year-old girl is brought to the clinic because of a painful and progressively enlarging axillary mass. Examination of a specimen previously obtained on biopsy showed hyperplastic glandular tissue without cytological atypia in the reticular dermis and subcutaneous tissue. Which of the following is the most likely diagnosis?

A) Accessory mammary tissue
B) Fibroadenoma
C) Juvenile hypertrophy
D) Juvenile papillomatosis
E) Lipoma

A

The correct response is Option A.

Accessory breast tissue typically occurs along the embryonic milk line and often enlarges during periods of hormonal stimulation, such as puberty. The rapid enlargement can be associated with pain. Histopathology shows glandular tissue and receptor staining is positive for estrogen and progesterone.

Fibroadenoma presents as a firm, rubbery nodule, usually within the ectopic breast, and histopathology shows epithelial and stromal proliferation.

Juvenile hypertrophy is progressive enlargement of the breasts during puberty, defined as greater than 3% of total body weight or greater than 3.3 lb (1500 g). Histopathology is similar to gynecomastia, with increased stromal collagen and fat.

A lipoma can present similarly to ectopic mammary tissue but histopathology would show mature adipose tissue.

Juvenile papillomatosis (“Swiss cheese disease”) was first described in 1980 and presents clinically similar to a fibroadenoma. Papillomatosis and epithelial hyperplasia, as well as sclerosing adenosis and cysts, are characteristic on histopathology. Approximately 10% of patients with juvenile papillomatosis will develop a malignancy. Although it occurs in pediatric patients, it is actually more common in adults.

How well did you know this?
1
Not at all
2
3
4
5
Perfectly
12
Q

A 35-year-old woman with symptomatic macromastia comes to the office to request bilateral reduction mammaplasty. She reports no history of breast biopsy or prior chest surgery. BMI is 25 kg/m2. Physical examination shows grade II ptosis with loss of upper pole fullness and good skin laxity, thus the decision is made to perform a bilateral reduction mammaplasty using a superomedial pedicle technique. Which of the following is the dominant blood supply to the nipple after this procedure?

A) Acromiothoracic artery
B) Lateral thoracic artery
C) Perforators of the internal thoracic artery
D) Posterior intercostal arteries
E) Superficial thoracic artery

A

The correct response is Option C.

The major blood supply from a superomedial pedicle to the nipple is the first through fourth perforators of the internal thoracic artery. The perforators originate from the first through fourth interspaces. The second perforating branch is considered the principal vessel.

While these vessels form anastomoses with branches from the lateral thoracic artery, the anastomoses are severed in the process of forming the superomedial pedicle.

The acromiothoracic artery does supply the skin arising along the free lower border of the pectoralis major muscle, but this is not involved in the perfusion of the nipple using the superomedial technique.

The posterior intercostal arteries have not been shown to reliably supply blood to the nipple-areola complex.

Although the superficial thoracic artery is similar in nature to the lateral thoracic artery in the branches supplying the nipple, the vessels are transected in the process of forming a superomedial pedicle.

How well did you know this?
1
Not at all
2
3
4
5
Perfectly
13
Q

A 52-year-old woman is scheduled for an oncoplastic reduction mammaplasty for ductal carcinoma in situ. The patient wants to keep the native nipple-areola complex, and the plan is to design the resection to maintain the blood supply to the nipple-areola complex. Which of the following arteries is the most common dominant blood supply to the nipple-areola complex?

A) Internal thoracic
B) Musculophrenic
C) Posterior intercostal
D) Superior intercostal
E) Thoracoacromial

A

The correct response is Option A.

Multiple cadaver studies have examined the blood supply to the nipple-areola complex (NAC). Most studies have found that the internal thoracic artery is the dominant blood supply, anastomosing with other arteries in the anterior fat.

In a study of diagnostic MRIs ultimately given a Breast Imaging Reporting and Data System (BI-RADS) I classification, review of the blood supply to the NAC demonstrated that 53.9% of the MRIs showed medial blood supply only to the NAC, and 42.30% had multi-zone, medial with lateral blood supply. A fresh cadaver study noted the dominant blood supply branches from the internal and external thoracic arteries, and another cadaver study demonstrated that the internal thoracic arteries are the primary source of NAC perfusion. The intercostal arteries do supply the breast, but have not been shown to be dominant. The thoracoacromial artery, a branch of the axillary artery, has four major divisions: pectoral, acromial, clavicular, and deltoid. The pectoral branch also supplies the breast, and anastomoses with the intercostal arteries and lateral thoracic arteries.

A study of women who underwent nipple-sparing mastectomy with preoperative MRI demonstrated that a dominant medial blood supply occurred 71.3% of the time, and a dual blood supply occurred almost 80% of the time. NAC necrosis was less likely to occur in patients with a dual blood supply to the breasts.

Understanding common anatomy and variants allows the surgeon to plan operative approaches. Intraoperatively, surgeons can verify adequate perfusion to the NAC by using indocyanine green fluorescence angiography or fluorescein infusion.

How well did you know this?
1
Not at all
2
3
4
5
Perfectly
14
Q

A 30-year-old woman comes to the office for follow-up one week after undergoing bilateral reduction mammaplasty. She has no other medical history. Preoperative pregnancy test was negative. On examination, both breasts appear swollen and edematous, and thick, milky discharge is expressed from the nipples. The patient denies fever or malaise. Which of the following laboratory values is most likely to be increased?

A) C-reactive protein
B) Estrogen
C) Human chorionic gonadotropin
D) Prolactin
E) White blood cells

A

The correct response is Option D.

The postoperative finding of milky discharge from both breasts suggests galactorrhea, a rare but known complication after breast surgery. The etiology of galactorrhea can be multifactorial, with stimulation of prolactin secretion as the pathophysiologic basis of the symptoms. Breast manipulation, periareolar incisions, and irritation of the fourth intercostal nerve have all been implicated as origins of the afferent signals to the pituitary gland, stimulating prolactin secretion. Treatment with bromocryptine is usually undertaken.

Estrogen levels and human chorionic gonadotropin levels would be increased in pregnancy and are less likely to be increased in this recent postoperative patient, assuming a preoperative pregnancy test was negative. The symptoms of pain, edema, and bilateral breast swelling in the absence of fever and redness should alert the surgeon to a diagnosis of galactorrhea; an increased prolactin level would be diagnostic. In the absence of infection, white blood cell count and C-reactive protein levels would be normal.

How well did you know this?
1
Not at all
2
3
4
5
Perfectly
15
Q

A 14-year-old girl comes to the office with a history of rapid significant increase in the size of her breasts with puberty. She wears a size 32H brassiere. The size of her breasts negatively affects her activities of daily living. Physical examination shows BMI is 21 kg/m2, and both breasts are enlarged with minimal asymmetry. Histology of the breasts is most likely to demonstrate a proliferation of which of the following types of tissue?

A) Adipose
B) Ductal
C) Lobular
D) Muscle
E) Stromal

A

The correct response is Option E.

The patient is presenting with juvenile (virginal) hypertrophy of the breast. In this patient, the growth of the breast is due to the hypertrophy of the stromal component of the breast tissue. In this case, the patient has a normal range BMI, decreasing the chances that the size of the breast is related to her weight.

The histology of the breast will demonstrate a predominance of stromal tissue. This is the connective tissue of the breasts, which includes the fibroblasts and fat. As noted, fatty tissue will be present in the breast; however, it is not a predominant component in true juvenile massive breast enlargement as compared with breast enlargement in the obese adolescent. Ducts will be present but not predominating, lobules will be absent or poorly formed, and muscle development is unrelated to breast size.

How well did you know this?
1
Not at all
2
3
4
5
Perfectly
16
Q

Which of the following arteries provides the blood supply to the superomedial pedicle in reduction mammaplasty procedures?

A) Internal mammary
B) Lateral third intercostal
C) Posterior fourth intercostal
D) Thoracoacromial
E) Thoracodorsal

A

The correct response is Option A.

The superomedial pedicle used in some reduction mammaplasty cases is supplied by arterial blood flow from the ipsilateral internal mammary artery and its intercostal branches. The other arteries are incorrect.

How well did you know this?
1
Not at all
2
3
4
5
Perfectly
17
Q

The dominant blood supply to the nipple-areola complex comes from which of the following arteries?

A) Anterior lateral intercostal
B) Internal mammary
C) Lateral thoracic
D) Superior epigastric
E) Thoracoacromial

A

The correct response is Option B.

Although the internal mammary artery, anterior intercostal arteries, lateral thoracic arteries, and thoracoacromial artery all supply the nipple-areola complex (NAC), the internal mammary artery provides the most consistent contribution, which has been confirmed in multiple cadaver studies as well as in vivo MRI studies. The superior epigastric artery arising from the internal mammary artery supplies the anterior abdominal wall.

Indeed, some authors have suggested that the decreased incidence of NAC necrosis with an inframammary fold incision when compared with a periareolar incision may be due to the preservation of the blood supply to the NAC using the former incision. The design of pedicles for reduction mammaplasty, similarly, has been historically informed by the blood supply to the NAC.

How well did you know this?
1
Not at all
2
3
4
5
Perfectly
18
Q

A 53-year-old woman who underwent periareolar mastopexy 13 years ago comes to the office to request reoperation of her now DD-cup–sized breasts. She wants improvement in the appearance of her breasts with greater projection and a decrease in her brassiere size to a B cup. Physical examination of the breasts shows flattened nipple-areola complexes that are 72 mm in diameter surrounded by circumferential hypertrophic surgical scars. The breasts are wide and bottomed out with a 14-cm distance between the inferior areolar border and the inframammary fold. Which of the following is the most appropriate technique to achieve the desired result?

A) Liposuction of the breasts with autologous fat transfer to the retroareolar region
B) Liposuction of the breasts with placement of breast implants
C) Liposuction of the lower poles of the breasts with excision of the hypertrophic areolar scars
D) Periareolar mastopexy with open excision of excess breast tissue
E) Wise pattern mastopexy with open reduction of excess breast tissue

A

The correct response is Option E.

The most appropriate technique to achieve this patient’s desired result of improved appearance of her breasts with increased projection and significantly decreased cup size is a secondary Wise pattern mastopexy with open reduction of her excess breast tissue. This technique will allow reduction in the diameter of the areola, give increased breast projection, and decrease the chance for recurrence of widened hypertrophic periareolar scarring.

Liposuction of the lower pole of the breast and periareolar scar revision could modestly decrease breast volume and possibly improve scar quality, but they would be ineffective at improving breast shape and projection, and in decreasing the areolar dimensions and excessive length of the lower pole of the breast.

Periareolar mastopexy with open reduction of excess breast tissue will not increase central breast projection or adequately address the excessive length of the inferior areolar to inframammary crease distance.

Liposuction of the breasts with placement of breast implants could improve central breast projection. This approach, however, would not provide the significant decrease in breast volume of three cup sizes which this patient desires, and would not improve the patient’s periareolar scars or the bottoming out of the lower poles of the breasts.

Liposuction of the breasts with fat grafting to the retro-areolar areas could increase central breast projection and decrease brassiere cup size, but it does not treat the hypertrophic areolar scarring or the abnormal lower pole dimensions of the breasts.

How well did you know this?
1
Not at all
2
3
4
5
Perfectly
19
Q

An otherwise healthy 52-year-old woman comes to the office for consultation for bilateral mastopexy. Her last mammogram 2 years ago was negative. Physical examination shows a palpable breast mass in the upper outer quadrant of the right breast that the patient has not noticed previously. Which of the following is the most appropriate next step in management?

A) Core needle biopsy
B) Diagnostic mammogram with ultrasound
C) Fine-needle aspiration biopsy
D) Mastopexy with open biopsy
E) Screening mammogram

A

The correct response is Option B.

The first step in the management of a newly found palpable breast mass is x-ray imaging to further characterize the tumor. The type of imaging required typically depends on the age of the patient at presentation. In females less than 30 years of age, ultrasound is typically the first (and possibly only) test ordered as the breast tissue is typically denser and mammography is not as effective. In women greater than 30 years of age, mammogram is usually the first test ordered. Mammography can evaluate both breasts for other incidental findings as well as further characterize the mass. Unless the results of the initial mammogram are definitive of a benign etiology of the mass, then an ultrasound is typically necessary as well. Ultrasound can distinguish cystic from solid masses and will help delineate the shape, borders, and acoustic properties of the mass. When the mass is suspicious, biopsy is guided by ultrasonography but this is typically not the initial treatment.

Mammography can be used for both screening and diagnosis. Screening mammography consists of two routine views, craniocaudal and mediolateral oblique, and is appropriate for asymptomatic patients. Diagnostic mammography incorporates additional views (e.g. tangential or spot compression views) in order to better delineate the area of concern. The current patient has a new finding of palpable mass on exam and requires a diagnostic mammogram for proper evaluation and management.

Given the patient’s age and presentation with newly palpable mass, x-ray imaging prior to any surgery is warranted to rule out malignancy. Proceeding with surgery that would rearrange the breast tissue may compromise the oncologic management of a possible breast cancer with incomplete excision and inability to obtain reliable margins that would require a completion mastectomy instead of the option for breast-conserving therapy.

How well did you know this?
1
Not at all
2
3
4
5
Perfectly
20
Q

A 54-year-old woman with a history of left mastectomy for breast cancer presents for right reduction mammaplasty for symmetry. In this patient, the incidence of occult breast cancer discovered incidentally in tissue specimens at the time of reduction mammaplasty is approximately which of the following?

A) 0.4%
B) 1%
C) 5%
D) 15%
E) 23%

A

The correct response is Option C.

There have been multiple studies on the incidence of breast cancer discovered in reduction mammaplasty specimens. The incidence of occult cancer detected in reduction mammaplasty specimens is typically very low (0.06 to 5.45%) but varies depending on the patient’s age and history of breast cancer. One specific study compared women undergoing reduction mammaplasty for symptomatic macromastia with women undergoing reduction mammaplasty for symmetry after mastectomy with or without reconstruction. Incidentally discovering occult breast cancer was much higher in women undergoing symmetry procedures (5.5 vs. 0.4%) versus those undergoing reduction mammaplasty for symptomatic macromastia. The important distinction in this clinical vignette is that the woman has had a mastectomy for breast cancer, and highlights several important points including:

The importance of a thorough history before reduction mammaplasty

Preoperative clinical examination

Screening mammography prior to the reduction mammaplasty

Pathologic examination of reduction mammaplasty specimens

Based on multiple studies, the other percentages listed are either too high or too low.

The treatment of occult cancers discovered during reduction mammaplasty depends on several factors including family history and evaluation of surgical margins.

How well did you know this?
1
Not at all
2
3
4
5
Perfectly
21
Q

A 43-year-old woman comes to the office for consultation for reduction mammaplasty. She wears a DDD brassiere and wants her brassiere size to be decreased to a C cup. Current medications include oral contraceptive pills. She does not smoke cigarettes. Height is 5 ft 5 in (167 cm) and weight is 145 lb (65.7 kg). BMI is 23.8 kg/m2. Physical examination shows the suprasternal notch to nipple distance is 29 cm and inframammary fold to nipple distance is 16 cm. Regardless of technique, which of the following factors is most likely to result in an increased risk for postoperative fat necrosis in this patient?

A) Massive weight loss
B) Oral contraceptive use
C) Patient age
D) Suprasternal notch to nipple distance
E) Tissue resection weight

A

The correct response is Option E.

Fat necrosis is one of the more common complications associated with reduction mammaplasty. Regardless of technique, the rates of fat necrosis have been reported in the 2 to 10% range. Fat necrosis presents as firm, soft-tissue masses that usually resolve spontaneously. It can be associated with redness and mild discomfort and may be confused with an infectious process. The literature is inconsistent with respect to detailed cause and effect or definitive correlations between fat necrosis and risk factors. However, some are generally agreed upon as significant and are mostly consistent in studies. Some of these risk factors for the development of fat necrosis include greater BMI, larger resection weights (both absolute and controlled for preoperative breast size), and long suprasternal notch to nipple distance (especially over 37 cm). Other factors that less clearly affect the rate of fat necrosis are smoking and the surgical technique employed.

Multiple studies have demonstrated increased risk of fat necrosis with greater obesity.

Although there have been some studies that suggest a correlation between fat necrosis and age, the studies have not reached a consensus nor have they shown statistical significance when evaluated in a controlled fashion. One study by Shermak et al. looked specifically at age-related risks and was not able to find a significant correlation with fat necrosis. There have been no studies or associations found to link exogenous hormone therapy or oral contraceptive use to increased fat necrosis. Alternatively, there has been some evidence to suggest that hormone supplementation might decrease the rate of infectious complications. The rate of fat necrosis and complication in general increases with longer suprasternal notch to nipple distances, most notably greater than 37 cm.

Massive weight loss is not associated with an increased risk for fat necrosis in the breast.

How well did you know this?
1
Not at all
2
3
4
5
Perfectly
22
Q

In a Wise-pattern reduction mammaplasty using the superior medial pedicle, sensation is provided to the nipple-areola complex by which of the following nerves?

A) Lateral cutaneous branch of the fourth intercostal nerve
B) Lateral cutaneous branch of the second intercostal nerve
C) Medial pectoral nerve
D) Terminal branches of the fourth and fifth anterior intercostal nerves
E) Terminal branches of the second and third anterior intercostal nerves

A

The correct response is Option D.

The nipple-areola complex is innervated by the lateral cutaneous branch of the fourth intercostal nerve as well as the terminal branches of the fourth and fifth anterior intercostal nerves. However, when a superior medial pedicle in a reduction mammaplasty is used, the contribution from the lateral branch of the fourth intercostal nerve is excised. The lateral cutaneous branch of the second intercostal nerve is also known as the intercostobrachial nerve, which provides sensation to the medial and posterior upper arm. The medial pectoral nerve innervates portions of the pectoralis major and minor.

How well did you know this?
1
Not at all
2
3
4
5
Perfectly
23
Q

A 44-year-old woman, gravida 4, para 3, is evaluated because of symptomatic macromastia. Bilateral reduction mammaplasty is planned. The patient’s mother was diagnosed with postmenopausal breast cancer at 53 years of age; the patient underwent genetic testing which was negative for BRCA mutation. Physical examination shows the patient wears a size 32F brassiere and has grade III ptosis, shoulder grooving, dense breast tissue without palpable masses or nipple discharge, and intertrigo. According to current American Cancer Society recommendations, which of the following breast imaging methods should be used before the planned reduction mammaplasty in this patient?

A) Diagnostic mammography
B) MRI
C) Screening mammography
D) Thermography
E) Ultrasonography

A

The correct response is Option C.

The American Society of Plastic Surgeons participates in the Choosing Wisely campaign, which advocates for evidence-based guidelines in determining diagnostic and therapeutic interventions.

The American College of Surgeons’ recommendations for breast cancer screening in average-risk, asymptomatic women are for an opportunity for a baseline mammogram at 40 to 44 years of age, annual screening from 45 to 54 years of age, and biennial screening for women older than 55 years of age who are in good health and have a life expectancy of at least ten years.

This patient is asymptomatic, and is not a BRCA carrier, and meets criteria for a screening mammogram.

A diagnostic mammogram is performed to evaluate abnormalities found on screening mammogram, in the context of breast cancer history, or with physical exam findings such as a breast mass, nipple discharge, or breast pain.

MRI is recommended as an adjunct to mammography in the case of a known BRCA mutation, if the first-degree relative is known to have the BRCA mutation but the patient is untested, or if there is a lifetime risk of 20 to 25% of breast cancer.

Thermography uses an infrared camera to show patterns of blood flow and heat on the surface of the breast. It is not a replacement for mammography, not recommended as part of screening protocols, and is not associated currently with any quality studies that demonstrate that it can be used effectively as a screening tool for breast cancer.

Ultrasonography is also used as an adjunct to screening mammography.

It is important to note that different societies have different recommendations on timing of screening mammography. The ACS recommends screening at 45 years of age, the American College of Radiology recommends screening starting at 40 years of age, and the USPSTF recommends biennial mammograms between 50 and 74 years of age.

How well did you know this?
1
Not at all
2
3
4
5
Perfectly
24
Q

A healthy 42-year-old woman undergoes bilateral reduction mammaplasty for symptomatic breast hypertrophy. Pathologic analysis of one of the tissue specimens shows ductal carcinoma in situ. Which of the following percentages best represents the incidence of this finding in a reduction mammaplasty specimen?

A) 1%
B) 7%
C) 10%
D) 12%
E) 25%

A

The correct response is Option A.

The histologic finding is consistent with a diagnosis of ductal carcinoma in situ (DCIS). The incidence of occult breast cancer in reduction mammaplasty specimens most closely approximates 1%. Invasive ductal carcinoma is the most common malignant lesion identified, but DCIS, lobular carcinoma in situ, Paget disease, and fibrosarcoma have also been reported less frequently. All breast tissue removed from women older than 40 years should be sent to pathology for microscopic analysis.

How well did you know this?
1
Not at all
2
3
4
5
Perfectly
25
Q

A 30-year-old woman with symptomatic macromastia is scheduled for a Wise pattern breast reduction. Which of the following postoperative complications is most likely in this patient?

A) Delayed wound healing
B) Fat necrosis
C) Hematoma
D) Hypertrophic scarring
E) Tear-drop deformity of the nipple-areola complex

A

The correct response is Option A.

In the 2005 prospective, multicenter trial of 179 patients by Cunningham, Gear, Kerrigan and Collins, reduction mammaplasty had an overall complication rate of 43%. The most common complication was delayed wound healing (21.6%), followed by spitting sutures (9.2%), hematoma (3.7%), nipple necrosis (3.6%), hypertrophic scars (2.5%), fat necrosis (1.8%), seroma (1.2%), and infection (1.2%).

Delayed wound healing correlated directly with average preoperative breast volume, average resection weight per breast, and smoking; and inversely with patient age. In this study, vertical techniques had a higher overall rate of complications.

In the Stevens, et al. report of their 11-year experience with outpatient breast reduction, delayed wound healing was also the most common complication.

How well did you know this?
1
Not at all
2
3
4
5
Perfectly
26
Q

Compared with reduction mammaplasty excisional techniques, which of the following is characteristic of liposuction-only reduction mammaplasty?

A) Does not typically impair breast-feeding potential
B) More effectively treats ptosis
C) Poses a higher risk to the blood supply of the nipple-areola complex
D) Results in a higher incidence of impaired sensation to the nipple-areola complex
E) Results in minimal swelling postoperatively

A

The correct response is Option A.

Liposuction-only reduction mammaplasty does not involve the use of a pedicle to ensure blood supply to the nipple. The nature of liposuction involves leaving major vessels and nerves intact and therefore does not pose an increased risk for blood supply loss to the nipple-areola complex. In addition, sensation to the breast as well as to the nipple-areola complex is not typically impaired following liposuction-only reduction mammaplasty. For similar reasons, breast-feeding potential is not typically compromised. The recovery from liposuction-only reduction mammaplasty can be quite significant, and it can take about 6 weeks for bruising and swelling to decrease and about 6 months for the breast to soften and for lumpiness to settle. Liposuction-only reduction mammaplasty does not work well when breast tissue is mostly glandular and thus has limited usefulness in those patients in whom it may seem most desirable, such as teenagers. Liposuction-only reduction mammaplasty does not more effectively treat ptosis.

How well did you know this?
1
Not at all
2
3
4
5
Perfectly
27
Q

A healthy 27-year-old woman is evaluated 16 weeks after bilateral reduction mammaplasty with an inferior pedicle technique. Histologic examination of the resected tissue shows no malignancy. She reports a tender mass in the right breast that she noted 8 weeks after operation. She is now apprehensive because her mother had breast cancer at age 34. Physical examination of the affected breast shows a palpable, slightly tender, discrete, firm 2-cm subcutaneous mass beneath the upper areolar border. Examination shows no skin dimpling, nipple retraction, erythema, or edema. Which of the following is the most appropriate next step in management?

A) Conduct needle aspiration of the lesion

B) Inject triamcinolone acetonide suspension 20 mg into the mass

C) Proceed to surgery for removal of the mass

D) Refer the patient for an oncology consultation

E) Schedule ultrasonography and mammography of the affected breast

A

The correct response is Option E.

Postsurgical changes in the breast after reduction mammaplasty encompass a variety of physical and radiographic manifestations. On presentation of a breast mass after reduction mammaplasty, a diagnostic protocol is used to determine whether operative intervention is appropriate, to avert unnecessary biopsy and to avoid overlooking breast malignancy. Fat necrosis, oil cysts, fibrosis, organizing hematoma, calcifications and, rarely, concurrent malignancy should be considered in this patient. In the absence of acute phenomena characteristic of a wound infection, work-up should consist of mammography and ultrasonography of the affected breast. Combining the radiographic findings with echographic appearance of the mass will help to differentiate fat necrosis and other benign conditions from the more ominous malignant etiology. The mammographic appearance of fat necrosis ranges from completely undetectable to a spiculated density and clustered microcalcifications. Many authors believe that the calcifications of fat necrosis can be distinguished from those seen with breast malignancies. Ultrasonographic findings include a solitary cyst, heterogenous echogenicity, and microcalcifications. Any remaining doubt as to the biologic nature of the mass should then be pursued with needle or open biopsy of the mass. Injection of any agent into the mass before it is definitively diagnosed is contraindicated, as is surgical removal or observation without obtaining a confident exclusion of malignancy. Referral to an oncologist would be premature in this instance and would provoke an unnecessary level of patient anxiety.

How well did you know this?
1
Not at all
2
3
4
5
Perfectly
28
Q

A 56-year-old woman undergoes bilateral reduction mammaplasty. Eight hundred grams per breast is removed and sent to permanent pathology. Which of the following results most likely requires further discussion with a breast surgeon?

A) Apocrine metaplasia
B) Atypical lobular hyperplasia
C) Fibroadenoma
D) Papillomatosis
E) Sclerosing adenosis

A

The correct response is Option B.

All of the answers are examples of benign breast disease. However, atypical lobular hyperplasia (ALH) is associated with an increased risk for breast cancer. Depending on other patient risk factors, chemoprevention with anti-estrogen medications and increased surveillance may be recommended.

How well did you know this?
1
Not at all
2
3
4
5
Perfectly
29
Q

A 47-year-old woman undergoes bilateral reduction mammaplasty surgery. Pathologic analysis of the resected breast tissue shows atypical ductal hyperplasia in the left breast. Which of the following is the most appropriate next step in management?

A) Gail model risk assessment
B) Hormonal therapy with aromatase inhibitors
C) Hormonal therapy with selective estrogen receptor modulators
D) Left-sided mastectomy
E) Postoperative radiation therapy

A

The correct response is Option A.

Atypical ductal hyperplasia is a risk factor for breast cancer in both the ipsilateral and contralateral breast, although the risk for the ipsilateral breast is higher. There is an approximately three-fold to five-fold increase in the risk for breast cancer in patients who have had biopsy-proven atypia. In an excisional biopsy, no additional surgery is needed for a finding of atypia; this is in distinction to atypia found on core biopsy, in which excision is recommended. In neither case is mastectomy indicated.

Radiation therapy is indicated for positive surgical margins in breast cancer excision, tumor size greater than 5 cm, more than four positive axillary nodes, and T4 disease.

The finding in this scenario should prompt the practitioner to perform a full risk assessment. Although the Gail model has been criticized for underestimating the risk for cancer in the context of atypical hyperplasia, it is still the most appropriate choice of the options listed. If the risk for breast cancer is sufficiently high after all factors are considered, hormonal therapy as a preventive measure may be indicated.

How well did you know this?
1
Not at all
2
3
4
5
Perfectly
30
Q

A 45-year-old woman, gravida 3, para 3, undergoes reduction mammaplasty. Pathologic examination of excised tissue shows a completely excised, 1-cm papilloma. Which of the following is the most appropriate next step in management?

A) External radiation therapy
B) Mammography
C) Sentinel lymph node biopsy
D) Subcutaneous mastectomy
E) Tamoxifen therapy

A

The correct response is Option B.

The management of papillomas found on breast core needle biopsy specimens is controversial. The concern is malignancy, and some institutions have reported false-negative rates in biopsy. The presence of atypia is an indication for complete excision, and is also associated with a final upstaging to in situ or invasive carcinoma. For these reasons, excisional biopsy is recommended for lesions found on core needle biopsy. In this case, the lesion was completely excised, which is the most aggressive treatment. Because the final pathology was benign disease, this excision is adequate.

A subcutaneous mastectomy is not necessary, and a sentinel lymph node biopsy is not indicated because the pathology is benign. Similarly, tamoxifen treatment and external radiation therapy are not indicated for this benign condition.

How well did you know this?
1
Not at all
2
3
4
5
Perfectly
31
Q

A 45-year-old woman comes to the office because she is interested in having reduction mammaplasty. She asks if undergoing reduction mammaplasty would decrease her risk of breast cancer. Which of the following is the most appropriate response to this patient?

A) The rate of breast cancer has been shown to be slightly increased in patients who undergo reduction mammaplasty
B) Reduction mammaplasty appears to decrease the rate by 90%
C) Reduction mammaplasty decreases the rate of breast cancer but less than prophylactic mastectomies
D) There is no evidence that reduction mammaplasty decreases cancer risk

A

The correct response is Option C.

Over the past decade, several large retrospective studies have looked at the rate of developing breast cancer after reduction mammaplasty. The breast cancer rates in patients undergoing reduction mammaplasty have consistently been decreased by about 30%. This differs from prophylactic mastectomy, which lowers the rate by as much as 90%.

32
Q

An otherwise healthy 17-year-old nulliparous girl is evaluated because of significant breast asymmetry. She has noted an increase in the size of her left breast over the past 2 months. Current medication includes a combined oral contraceptive. The patient undergoes mammography and biopsy. Pathologic examination of excised tissue shows a benign phyllodes tumor. Which of the following is the most appropriate next step in management?

A) Enucleation
B) Mastectomy with sentinel lymph node biopsy
C) Progestin-only oral contraceptive
D) Tamoxifen followed by lumpectomy
E) Wide local excision

A

The correct response is Option E.

The three most common causes for adolescent unilateral breast enlargement are giant fibroadenoma, phyllodes tumor (previously called cystosarcoma phyllodes), and juvenile breast hyprterophy. Differentiation between phyllodes tumor and giant fibroadenoma on core needle biopsy is difficult. Phyllodes tumors are fibroepithelial tumors and stromal derived. The stromal component can appear similar to a fibroadenoma, and this similarity can make the two difficult to distinguish; in some cases, the stromal component resembles a soft-tissue sarcoma. Core needle biopsy is performed for diagnosis, and phyllodes tumors typically have increased cellularity, mitosis, and stromal overgrowth when compared with fibroadenomas.

Phyllodes tumors of the breast represent approximately 3% of breast neoplasms. They are classified as benign, borderline, or malignant. Wide excision with 1-cm margins is recommended for all classifications. The extent of resection is determined by the grade, and the grade is associated with the risk of local recurrence.

Margin-negative, breast-conserving therapy is appropriate for benign phyllodes tumors. Malignant phyllodes tumors behave more similarly to sarcomas than to other types of breast cancer. Although there is controversy about the role of radiation therapy, this modality is less effective than surgery, and is reserved for margin-positive, malignant tumors. Chemotherapy is not indicated in this patient. The greatest risk with benign phyllodes tumors is local recurrence, and overall, the prognosis of these tumors is considered excellent.

The only known condition associated with the development of phyllodes tumors is Li-Fraumeni syndrome.

33
Q

A 16-year-old girl comes to the office with her parents because she would like reduction mammaplasty surgery. Height is 5 ft 1 in (155 cm) and weight is 160 lb (72.6 kg). She wears a size 36G brassiere and her breast size has remained the same for the past year. She has significant physical manifestations of macromastia. Her parents report that she does not want to go to work or school because she is embarrassed by the size of her breasts. Which of the following is the most appropriate management of this patient’s condition?

A) Defer reduction mammaplasty until the patient has lost at least 20 lb (9.1 kg)
B) Defer reduction mammaplasty until the patient is at least 22 years of age
C) Perform reduction mammaplasty
D) Refuse to perform surgery until the patient is evaluated by a psychiatrist

A

The correct response is Option C.

Breast hypertrophy can affect girls as young as 10 to 15 years old and can result in massive breast development that can have profound physical and psychological impact on the patient. Reduction mammaplasty is indicated for these patients, despite the risk of breast growth postoperatively, which may necessitate a secondary operation. As long as the patient has a mature attitude, understands the permanence of the scars involved, and has parents who are supportive of her decision, then surgery should not be delayed simply to wait until the patient reaches a certain age. The large breasts are already a major problem and further growth will compound the problem and make it difficult to manage later.

With the rise in childhood obesity, there has been an increase in the number of patients seeking adolescent reduction mammaplasty. The cause of macromastia in pubertal and parapubertal girls is variable and includes endocrine changes, childhood obesity, and juvenile (virginal) hypertrophy of the breast. Benefits of reduction mammaplasty include resolution of pain, improved quality of life, extroversion, and emotional stability.

Reduction mammaplasty has been clearly proven to decrease the physical manifestations associated with macromastia, regardless of height and weight, as well as provide psychosocial benefits and improve self-esteem, regardless of age of patient.

34
Q

Reduction mammaplasty using which of the following pedicles has the greatest risk of altered nipple sensation?

A) Inferior
B) Inferocentral
C) Lateral
D) Superior
E) Superomedial

A

The correct response is Option D.

Reduction mammaplasty with a superior pedicle that involves resection of the tissue at the base of the breast is associated with a higher risk of injury to the nerve branches that innervate the nipple-areolar complex. Innervation from the lateral cutaneous branches runs deep within the pectoral fascia before sharply turning in an anterior direction to innervate the nipple from its deep aspect.

The lateral pedicle, inferior pedicle, and inferocentral pedicle save the tissue containing the lateral cutaneous branches, decreasing the chance for injury to this nerve and reduced nipple sensation. The superomedial pedicle preserves the anterior cutaneous branches which run superficially from the medial aspect of the breast, and also provide sensation to the nipple. The superior pedicle resects both the medial and lateral innervations to the nipple.

35
Q

An otherwise healthy 44-year-old woman comes to the office for reduction mammaplasty consultation. She wears a size 44DD brassiere. Physical examination shows rashes underneath the breasts, shoulder grooving from brassiere straps, and shoulder pain. A reduction is planned with removal of 500g of tissue bilaterally. The woman reveals that her sister underwent reduction mammaplasty with a much larger resection size and questions whether she will have the same relief of symptoms. Which of the following is the most appropriate response?

A) Larger reductions are associated with less marked relief of symptoms
B) Larger reductions are associated with more marked relief of symptoms
C) Smaller reductions are associated with less marked relief of symptoms
D) Smaller reductions are associated with more marked relief of symptoms
E) Resection size is not associated with relief of symptoms

A

The correct response is Option E.

The ASPS has a clinical guideline summary on reduction mammaplasty, based on the available evidence. Although insurance companies often use resection weight as a criterion for coverage, resection weight is not necessarily associated with relief of symptoms; thus, predictions of relief of symptoms must be made based on the individual clinical picture. This evidence is graded B.

The risk of complications, however, does increase with resection weight (Grade B evidence). This risk of complications must be weighed against the potential for relief of symptoms with large resection weights. A distinction is made between resection weight and BMI. The ASPS guideline found only “inconclusive” data on the association between BMI and the risk of complications.

36
Q

Which of the following best describes the role of estrogen in breast function?

A) Decreases cell division
B) Decreases fibrocystic changes
C) Facilitates ductal growth
D) Facilitates glandular growth
E) Facilitates periductal stromal development

A

The correct response is Option C.

Breast development is a complex interplay of multiple factors. Estrogen and progesterone play a significant role in breast development—not only at puberty, but also during and after pregnancy, and during and after menopause. In general, estrogen causes ductal proliferation, while progesterone causes glandular proliferations. Progesterone is similarly responsible for periductal stromal development. Estrogen increases, not decreases, cell division, and is also associated with increased, not decreased, fibrocystic changes.

37
Q

A 5-ft 1-in (155-cm), 185-lb (84-kg), 45-year-old woman comes to the office for follow-up 1 week after she underwent reduction mammaplasty with a medial pedicle in the outpatient facility. BMI is 35 kg/m2. On examination, the right nipple-areola complex is dusky and cool. Nipple ischemia is suspected. Which of the following is the most appropriate next step in management?

A) Debridement of the necrotic nipple with primary closure
B) One-stage debridement of the necrotic nipple and reconstruction of the nipple-areola complex
C) One-stage exploration and conversion to a free nipple graft
D) Release of all insetting sutures
E) Observation only

A

The correct response is Option E.

Partial or total nipple necrosis may be one of the most devastating complications of reduction mammaplasty. The incidence of compromise of the nipple-areola complex is typically less than 5% after breast reduction. Increased BMI is a risk factor for and increased risk of both nipple necrosis and wound healing complications. Reduction mammaplasty is frequently performed on an outpatient basis. Patients are seen within a week of surgery for wound checks, but this may be too late to address a nipple with vascular compromise.

If nipple ischemia is noted at the time of surgery during inset, stitches should be released and vascularity reevaluated. An objective assessment of blood flow can be aided with the use of fluorescein injection and a Woods lamp, or with newer screening modalities that are currently being evaluated for this purpose. While inset may be reattempted, the patient will likely require conversion to a free nipple graft. The nipple should be grafted to well-vascularized, deepithelialized dermis and not to ischemic fat that may be part of the compromised pedicle.

If nipple ischemia is detected in the early postoperative period, and there is no hematoma or issue with external compression, the nipple should be released from its inset position. This will relieve tension on the pedicle. If the nipple does not improve, the patient is taken back to the operating room for free nipple grafting.

If nipple ischemia is not identified in the early postoperative period, the patient should be treated with conservative wound care until healing is complete. Nipple reconstruction can then be undertaken in a delayed manner.

38
Q

A 45-year-old woman comes to the office for consultation regarding reduction mammaplasty because of pain of the neck and upper back. She currently wears a size 42 DDD brassiere and would like to be a C cup. A vertical reduction mammaplasty with a superomedial pedicle is planned. Which of the following is the dominant blood supply for this pedicle?

A) Ascending branch from the fifth intercostal space
B) Ascending branch from the sixth intercostal space
C) Descending branch from the first intercostal space
D) Descending branch from the second intercostal space
E) Descending branch from the fourth intercostal space

A

The correct response is Option D.

Almost 60% of the blood flow to the breast is from the internal mammary artery. The second and third anterior perforating branches are most dominant. In a superomedial pedicle both the second and third descending branches are captured in the pedicle. In a pure medial pedicle, it is usually the third. The other branches do not constitute any significant contribution to the pedicle blood supply in a superomedial or medial reduction.

39
Q

A 16-year-old girl is referred to the office because of an 8-month history of sudden and rapid enlargement of the right breast. She reports no other symptoms. Physical examination shows a large, palpable mass on the lower half of the right breast. Marked nipple-areola complex stretching, prominent dilated veins, and skin ulceration inferolateral to the mass are noted. Mammography and ultrasonography show a dense, circumscribed, homogenous 8-cm mass in the right breast. Which of the following is the most likely diagnosis?

A) Carcinoma
B) Cyst
C) Giant fibroadenoma
D) Juvenile breast hypertrophy
E) Phyllodes tumor

A

The correct response is Option C.

This patient has a fibroadenoma, the most common breast neoplasm in adolescent females. Giant fibroadenomas are typically solitary, firm, nontender, and symptoms include a rapid asymmetric breast enlargement with prominent overlying veins and occasional pressure-induced skin ulceration. These lesions are larger than 5 cm and occur at or soon after the onset of puberty. These lesions are typically treated with enucleation using reduction mammaplasty techniques for optimal symmetry with the contralateral breast. Mastectomy is not indicated, and no other adjuvant therapy is necessary. Smaller fibroadenomas may be watched conservatively, with minimal risk of malignant transformation. Surgical intervention is indicated in cases of mastodynia, neck/back pain secondary to large size, difficulty with clothing due to asymmetry, and to alleviate patient concern.

Carcinoma would be unlikely in this age demographic. The differential diagnosis would additionally include cystic enlargement, breast hypertrophy, or phyllodes tumor.

Juvenile breast hypertrophy may occur as unilateral or bilateral breast enlargement. The enlargement is diffuse without evidence of a discrete mass or nodularity. Juvenile breast hypertrophy typically occurs in early puberty, rarely regresses spontaneously, and is much more severe than simple breast hypertrophy. The underlying cause is attributed to estrogen stimulation at the onset of the first menses. Treatment is reduction mammaplasty.

Phyllodes tumors are large, benign tumors that typically occur in the perimenopausal patient. They are histologically distinct from fibroadenomas, and transformation of a fibroadenoma to a phyllodes tumor is exceptionally rare.

40
Q

A 37-year-old woman comes for evaluation of symptomatic macromastia after failure of conservative treatment. Height is 5 ft 4 in (163 cm) and weight is 245 lb (111 kg). BMI is 42 kg/m2. Physical examination shows Grade II ptosis and symmetrical macromastia. The estimated weight of tissue resection is 1200 g per side. An inferior pedicle reduction mammaplasty is planned. Which of the following places this patient at greatest risk for postoperative hematoma?

A) Hypotensive general anesthesia
B) Obesity
C) Omitting closed suction drains
D) The patient’s age
E) Weight of the resected specimen

A

The correct response is Option A.

The risks associated with reduction mammaplasty include local complications such as healing problems, nipple necrosis, loss of nipple sensitivity, infection, hematoma, hypertrophic scarring, fat necrosis, and asymmetry; systemic effects include deep venous thrombosis, pulmonary embolism, atelectasis, and a number of other surgical and anesthesia-related complications. The quantified risk of complications resulting from reduction mammaplasty increases with the weight of the resected specimen. These include wound-healing problems and nipple sensitivity. Some authors have recommended that normotensive anesthesia be administered throughout the procedure. While local and systemic risks are associated with elevated BMI, the degree of obesity does not demonstrate a correlation between risks and elevated body weight. The rate of hypertrophic scarring was shown to decrease with larger resections, possibly because of relief of skin tension or the presence of attenuated dermal thickness. Intraoperative hypotension, utilized to diminish blood loss during surgery, results in a higher rate of subsequent hematoma. Randomized studies documenting the risk of hematoma have shown no difference between patients with and without the use of closed suction drains.

41
Q

A 15-year-old girl is brought to the office because her breasts have enlarged rapidly. She says she has severe back pain and posture problems. Her parents report that her brassiere size increased from 34B to 34E at age 13 years. There has been no increase in size for the past 12 months. Height is 5 ft 5 in (165 cm) and weight is 140 lb (63 kg). Which of the following is the most appropriate management?

A) Antiestrogen hormone therapy
B) Diet and exercise program to lose 15 lb (6.8 kg)
C) Reduction mammaplasty
D) Six-month testosterone injection protocol
E) Observation until the patient is age 18 years

A

The correct response is Option C.

The accurate diagnosis of benign pediatric breast tumors is essential for proper treatment. When bilateral enlargement that is grossly out of proportion occurs at menarche, it is termed juvenile hypertrophy. After the enlargement has stabilized for approximately 1 year, the treatment is surgical. There is only anecdotal evidence for using antiestrogen hormone therapy (Tamoxifen). Proper diet and exercise would be expected to reduce the size slightly, but the usual resection in these cases is 1800 g or more.

The use of testosterone injection is not indicated for juvenile hypertrophy.

Observation, while indicated at first to allow the process to be fully manifested, would be appropriate after the size had stabilized for about a year. Then it would be appropriate for the surgeon to reduce the size if necessary. The differential diagnosis for pediatric breast enlargement includes fibroadenomas, phyllodes tumor, and cancer.

42
Q

An 8-year-old girl is brought to the office by her mother because her daughter?s breasts have begun to develop. The mother says that her daughter has no history of serious illness. Height is 4 ft 2 in (127 cm, in 50th percentile) and weight is 55 lb (25 kg, in 50th percentile). Physical examination shows that her current cup size is a B. No pubic hair or vaginal mucosal thickening is noted. Which of the following is the most likely diagnosis?

A) Benign premature thelarche
B) Cushing syndrome
C) Gynecomastia
D) McCune-Albright syndrome
E) Precocious puberty

A

The correct response is Option A.

Knowledge of normal puberty and abnormal conditions of puberty is critical for any plastic surgeon evaluating the pediatric breast.

Tanner staging requires evaluation of pubic hair (males and females), genitals (males), and breasts (females). Because the patient described lacks pubic hair (Tanner I) and vaginal mucosal thickening, she has not started puberty in other areas. This excludes precocious puberty. If breasts develop before puberty has begun in other areas, it is considered benign premature thelarche and requires no intervention.

Gynecomastia is a hyperplastic condition of breast tissue. The patient described does not have hyperplastic breasts. In one series, hyperplastic breast abnormalities such as gynecomastia were the most common indication for operative intervention, with an average age of operation in the late teens.

McCune-Albright syndrome, also known as polyostotic fibrous dysplasia, is a condition characterized by premature puberty. Patients often begin menstruation before breast development. Patients also develop bony abnormalities, gigantism, and café-au-lait spots.

43
Q

A 38-year-old woman, gravida 2, para 2, is scheduled to undergo reduction mammaplasty because of pain in the neck and shoulders. She wears a size 44E brassiere. Physical examination shows pendulous breasts. The sternal notch-to-nipple distance is 40 cm. Hypertrophy of which of the following muscles is most likely in this patient?

A) Latissimus dorsi
B) Levator scapulae
C) Pectoralis major
D) Rhomboid major
E) Trapezius

A

The correct response is Option E.

Each of the muscles described is an extrinsic muscle of the back. The trapezius elevates the scapula in squaring the shoulders, and the superior, middle, and inferior fibers act together to pull the scapulae posteriorly, bracing the shoulders. The latissimus extends, adducts, and medially rotates the humerus. The levator scapulae elevate the scapula and rotate the glenoid cavity inferiorly, and the rhomboid major and minor together retract the scapula and fix the scapula to the thoracic wall.

In mammary hypertrophy, the downward pull of the breasts rotates the shoulders forward, requiring significantly more work of the shoulder girdle muscles. It is the unique function of the trapezius to ?square? the shoulders that makes it most vulnerable in mammary hypertrophy; because of the extra work this requires, it often becomes hypertrophic.

Patients often complain of shoulder, neck, and upper back pain as a result of the trapezius strain.

44
Q

A 25-year-old woman is scheduled to undergo breast reduction with resection of 2.4 lb (1100 g) from each breast. Current weight is 200 lb (91 kg), and height is 5 ft 8 in (173 cm). Physical examination shows macromastia with nipple-sternal notch distance of 34 cm on the left and 35 cm on the right. A photograph is shown. Which of the following postoperative complications is most likely in this patient?

A) Bleeding
B) Fat necrosis
C) Infection
D) Seroma
E) Wound breakdown

A

The correct response is Option E.

Breast reduction is one of the most commonly performed procedures in plastic surgery, and outcomes following breast reduction have been well studied. Common risks associated with breast reduction include infection, symptomatic scar, seroma, wound healing complications, fat necrosis, asymmetry, and need for reoperation. BMI, volume of breast tissue resection greater than 1000 g per breast, and tobacco use are the greatest risk factors for complication following surgery. Wound healing complications are more likely to occur following breast reduction in a young, healthy, but obese patient with large resection volumes. Other complications might occur but are less common. None of the other complications have been associated with risk factors.

45
Q

A 43-year-old woman is undergoing bilateral reduction mammaplasty with the inferior pedicle technique. The dermis is preserved during deepithelialization of the pedicle to protect which of the following anatomical structures?

A) Perforators from the internal mammary artery
B) Perforators from the lateral thoracic artery
C) Sebaceous glands
D) Subdermal plexus
E) Superficial layer of the superficial fascia of the breast

A

The correct response is Option D.

The major blood supply to the breast comes from perforating branches of the internal mammary artery, lateral branches of the posterior intercostal arteries, and branches of the axillary artery. The blood supply from the axillary artery includes the pectoral branches, the highest thoracic artery, and the lateral thoracic artery. Those vessels from the pectoral branches enter underneath the muscle before coming through it to supply the breast tissue. The vessels from the lateral thoracic artery, known as the lateral mammary branches, wrap around the lateral border of the pectoralis muscle to supply the lateral breast. The second, third, and fourth perforating branches from the internal mammary artery, known as the medial mammary arteries, enter the medial aspect of the breast. The perforating branches from the second, third, and fourth posterior intercostal arteries, known as the mammary branches, enter the breast laterally. The vascular arcades seem to be concentrated at the periphery of the breast (the cutaneoglandular plexus), and the larger vessels appear to lie not far beneath the skin, superficial to the glandular tissue. This finding has led certain authors to conclude that resection of the gland should not commence fewer than 2 to 3 cm from the chest wall; if skin flaps are elevated, they should be kept at least 2 cm thick for maximum viability. This also justifies preservation of the dermis when deepithelializing flaps to protect the subdermal plexus from injury.

Perforators from the internal mammary artery and lateral thoracic artery supply the breast parenchyma. The superficial layer of the superficial fascia of the breast parenchyma does not provide vascularity.

46
Q

Which of the following arteries is the dominant blood supply to the nipple-areola complex?

A) Axillary
B) Internal mammary
C) Subclavian
D) Superficial epigastric
E) Thoracodorsal

A

The correct response is Option B.

Multiple studies have been performed to document the blood supply to the breast and the wide range of normal. The nipple-areola complex receives its blood supply from the mammary arteries, which are a branch of the subclavian artery. The subclavian artery becomes the axillary artery and gives off the thoracodorsal artery. The superficial epigastric artery supplies the upper abdomen. The mammary arteries are sometimes referred to as the thoracic arteries.

47
Q

A 16-year-old girl is referred by her pediatrician for mammaplasty because of breast hypertrophy that has worsened during the past 2 years. She wears a size 36DD brassiere and has constant pain in the shoulders and back due to the weight of her breasts. Menarche occurred at 10 years of age. Height is 5 ft 4 in (163 cm), and weight is 165 lb (75 kg). Physical examination shows breast hypertrophy, shoulder grooving, intertrigo dermatitis, and striae. An abnormality of which of the following is the most likely cause of this patient’s condition?

A) End-organ responsiveness to estrogen
B) Number of estrogen receptors
C) Progesterone concentration
D) Prolactin concentration
E) Serum estrogen concentration

A

The correct response is Option A.

Abnormal end-organ responsiveness to estrogen is the predominant factor leading to breast hypertrophy. It has been demonstrated that normal levels of estrogen, progesterone, and prolactin exist in patients with breast hypertrophy. These patients also have a normal number of estrogen receptors.

48
Q

A 33-year-old woman comes to the office because of a new lump in her right breast 6 weeks after undergoing bilateral reduction mammaplasty using the inferior pedicle technique. Preoperative examination of the breasts showed no abnormalities. Current examination shows a hard, nontender mass in the lateral aspect of the upper right breast. Which of the following is the most likely diagnosis?

A ) Abscess

B ) Fat necrosis

C ) Fibroadenoma

D ) Hematoma

E ) Seroma

A

The correct response is Option B.

A patient presenting with a hard, nontender lump 6 weeks after reduction mammaplasty is most likely to have fat necrosis. This is usually the result of vascular compromise to areas of the parenchyma associated with hemorrhagic necrosis. Drainage from fat liquefaction is often the first sign of infection. Cellulitis and fever may then result. Small areas of fat necrosis can be managed conservatively, and secondary revision can be performed after a period of 6 months to 1 year. If skin and fat necrosis is extensive and associated with an infection, surgical debridement and antibiotics are required. Secondary closure and grafting are required after the infection has resolved.

An abscess or hematoma would be firm but likely tender. Fibroadenoma would not likely be palpable so early postoperatively. However, if the lump does not resolve within a few weeks, CT scan or ultrasonography should be considered to rule out malignancy. Seroma would most likely have a softer consistency.

49
Q

An otherwise healthy 33-year-old woman comes to the office because of back pain, brassiere grooving, and an inframammary rash. She wears a size 42 L brassiere. She has one child and says she would like to have more children. BMI is 33 kg/m2. The distance from nipple to sternal notch is 37 cm bilaterally, and the distance from nipple to inframammary fold is 16 cm bilaterally. A photograph is shown. Which of the following is the most appropriate reduction mammaplasty technique for this patient?

A ) Inferior pedicle with Wise skin pattern

B ) Lower pole amputation with nipple grafting

C ) Medial pedicle with vertical skin pattern

D ) Superior pedicle with vertical skin pattern

E ) Superior pedicle with Wise skin pattern and nipple grafting

A

The correct response is Option A.

The patient described is a young, healthy woman, who may desire to have more children later in life, making nipple preservation an optimal choice. She is large in size and has ptosis, making an inferior pedicle with Wise pattern the best technique to ensure adequate tissue removal. This technique is also able to reduce the vertical dimension of the breast, which is more challenging with vertical skin pattern reduction mammaplasty. Inferior pedicle reduction mammaplasty can achieve significant reduction in breast volume and alleviate preoperative symptoms with a low risk of complications.

Nipple grafting techniques are unnecessary in this woman, whose nipple needs to be elevated only 13 to 16 cm, and who may desire to have another child in the future. Medial pedicle technique is not widely used. It has been described with limited incision techniques and in combination with Wise pattern for severe macromastia. Limited incision techniques might fail to achieve reliable reduction in breast volume and result in aesthetic dimensions as well. These vertical scar techniques are best reserved for smaller volume reductions.

50
Q

A 37-year-old woman obtains a baseline postoperative mammogram six months after undergoing reduction mammaplasty. The presence of which of the following calcifications is most likely to cause the plastic surgeon to order additional evaluation?

A ) Branching

B ) Dystrophic

C ) Eggshell

D ) Popcorn-like

E ) Rod-like

A

The correct response is Option A.

The development of microcalcifications on mammogram images after surgery requires careful analysis. Calcifications can be grouped into three categories: malignant, indeterminate, or benign. Malignant calcifications can appear as casting (linear and branching) or pleomorphic (granular). Benign calcifications have many appearances and include popcorn-like (fibroadenoma), large rod-like (secretory), round eggshell (oil cysts), and dystrophic or coarse (fat necrosis). The development of masses, architectural distortion, and skin thickening can be seen following surgery. A complete evaluation is suggested to determine if a biopsy is necessary. Tissue biopsy is recommended for calcifications that are determined to be indeterminate or malignant.

51
Q

A 26-year-old woman is scheduled to undergo reduction mammaplasty. In designing the pedicles for the procedure, which of the following arteries is the predominant blood supply to the breast?

A ) Internal thoracic

B ) Lateral thoracic

C ) Supreme thoracic

D ) Thoracoacromial

E ) Thoracodorsal

A

The correct response is Option A.

Knowledge of the blood supply to the breast is important to the design of the pedicle to the nipple-areola complex. This knowledge can also be used when designing pedicles for the reconstruction of a breast after prior surgery, such as a lumpectomy. Perforating branches off of the internal thoracic artery (also called the internal mammary artery) are the predominant blood supply to the breast. The lateral thoracic and thoracoacromial arteries also contribute to the blood supply of the breast but to a lesser degree. The thoracodorsal artery and supreme thoracic artery do not contribute to the blood supply of the breast to any significant degree.

52
Q

Which of the following is the main disadvantage of reduction mammaplasty by liposuction alone?

(A) Dissemination of occult cancer

(B) Inability to breast-feed

(C) Inadequate correction of ptosis

(D) Increased hematoma formation

(E) Persistence of back pain

A

The correct response is Option C.

Liposuction alone is one of many techniques available to reduce the symptoms of macromastia. The primary advantage of this technique is the lack of scars. Several authors cite other advantages such as rapid return to work and exercise, decreased operative time, normal sensation, and full ability to breast-feed. Complications such as hematoma, seroma, and nipple necrosis are minimal compared to incisional techniques, and reductions of one to two cup sizes are reported. Symptoms of macromastia are relieved. Authors warn that this technique is not effective in young patients with dense breast tissue and little fatty tissue.

Disadvantages of this technique include difficulty assessing the amount of breast tissue removed because of the infiltration of tumescent solution, lack of pathologic examination, inadequate tissue removal in large reductions, and poor skin shrinkage. Short-term studies of postoperative mammograms show occasional benign calcifications, but longer-term studies are still needed. Dissemination of cancer is a theoretical risk but has not been reported. The incidence of occult cancer in reduction mammaplasty patients is less than 0.5%. Although nipples will rise somewhat with liposuction, the average elevation is 2 to 6 cm, and patients still have nipples which are at or below the inframammary fold (first- or second-degree ptosis).

The preferred candidate for liposuction-only reduction is described as a young patient with good skin elasticity, minimal to moderate hypertrophy, and no ptosis.

53
Q

For reduction mammaplasty, which of the following is the greatest advantage of a vertical procedure over an inverted-T method?

(A) Decreased risk of hematoma

(B) Greater ability to breast-feed

(C) Increased sensation of the nipple

(D) Initial natural shape of the breast

(E) Smaller scar

A

The correct response is Option E.

Vertical mammaplasty is a technique that uses adjustable markings, an upper pedicle for the areola, and a central breast reduction with decreased undermining of the skin. Key features of the vertical scar reduction mammaplasty include skin excision in only one direction, which reduces scar burden, and central vertical glandular excision, which contributes to improved postoperative shape by narrowing the breast while maximizing projection as a result of suturing the medial and lateral pillars together. There is no wide periareolar skin excision; therefore, circumareolar scar quality is not compromised by excess tension.

However, this method is more intuitive and inherently less precise than the inverted €‘T method. Because of the central and posterior nature of glandular resection in vertical mammaplasty, it is more difficult to determine the end point of resection.

A recent study comparing vertical mammary reduction with the Wise pattern technique showed no statistical differences in the rate of hematomas and nipple numbness. Breast-feeding is possible with either technique but may require supplementation with formula. The initial shape in a vertical mammaplasty is poor; it has a flattened lower pole, and dog-ears are frequently present. This improves after three to six months, although small revisions are sometimes necessary.

54
Q

Which of the following is the estimated incidence of detection of occult breast cancer during reduction mammaplasty?

(A) 0.002%

(B) 0.02%

(C) 0.2%

(D) 2%

(E) 20%

A

The correct response is Option C.

The average risk of occult breast cancer in patients undergoing breast reduction has been estimated to be 0.27% (0.11%, 0.75%, 0.71%, 0.05%, 0.06%, 0.16%, and 0.09% in seven journal articles published between 1997 and 2006). Cancer is often found at the in situ or early stages. Definitive therapy depends on size, pathology, location, and status of margins. Consultation with an oncologist and further pathologic analysis are generally necessary.

55
Q

A 16-year-old girl is referred to the office by her pediatrician for reduction mammaplasty because of breast hypertrophy that has been worsening over the past two years. She wears a size 36DD brassiere and has constant pain in the shoulders and back from the weight of her breasts. Height is 5 ft 4 in, weight is 165 lb, and menarche was at 10 years of age. Physical examination shows juvenile breast hypertrophy, shoulder grooving, intertrigal dermatitis, and striae. Abnormality of which of the following is the most likely cause of this patient=s condition?

(A) End €‘organ responsiveness to estrogen

(B) Estrogen level

(C) Number of estrogen receptors

(D) Progesterone level

(E) Prolactin level

A

The correct response is Option A.

Abnormal end €‘organ responsiveness to estrogen is the predominant factor leading to breast hypertrophy. It has been demonstrated that normal levels of estrogen, progesterone, and prolactin exist in patients with juvenile breast hypertrophy. These patients also have a normal number of estrogen receptors.

56
Q

A 38 €‘year €‘old woman comes to the office for consultation regarding reduction mammaplasty. She currently wears a size 36DD brassiere and wants to be able to wear a C cup brassiere postoperatively. Height is 5 ft 6 in and weight is 120 lb. On physical examination, the distance from nipple to sternal notch is 35 cm. Selection of a vertical mammaplasty technique in this patient is limited by which of the following?

(A) Desired postoperative cup size

(B) Fat content of the breasts

(C) Lateral positioning of the nipples

(D) Length of the pedicle

(E) Preoperative cup size

A

The correct response is Option D.

The contraindications to performing a vertical reduction mammaplasty are the length of the pedicle and the amount and quality of the remaining skin. Blood supply to the nipple may be compromised in a very long pedicle. It is impossible to give a numeric value to this length. Lassus recommends a transposition of no more than 9 cm.

Vertical reduction mammaplasty more easily achieves consistently high €‘quality results when the method is applied to a small or moderate reduction (< 800 g per side). Although reductions of more than 1000 g are technically difficult to achieve, they are not impossible and have been reported by some centers. With the adjunct of the L €‘shaped scar, larger amounts of skin can be resected, limiting the need for postoperative revisions.

The lateral positioning of the nipples is not a contraindication to vertical reduction mammaplasty. Although Lejour reported delayed wound healing in cases with high fat content of the breast, this does not preclude the vertical method.

57
Q

During a reduction mammaplasty procedure, preservation of sensation in the nipple €‘areola complex is most dependent on which of the following?

(A) Amount of tissue resected

(B) Nipple-to-notch distance

(C) Pedicle location

(D) Preoperative breast volume

(E) Skin incision used

A

The correct response is Option C.

The most important determinant in preserving sensation in the nipple €‘areola complex is the anatomic location of glandular resection. Superior glandular pedicle techniques with tissue resections at the base of the breast are associated with higher risk of injury to the nerve supply. Lateral, inferior, and medial-based pedicles allow for better preservation of the nerve supply.

The amount of tissue resected has not been shown to have a statistically significant effect on preservation of sensation in the nipple-areola complex.

Preoperative breast volume is a factor in preoperative nipple-areola sensation (ie, the sensitivity of the nipple-areola decreases as the breast volume increases). However, preoperative breast volume, type of skin incision used, or nipple-to-notch distance have not been found to be the most important factor in preservation of nipple sensation.

58
Q

A 38-year-old woman has severe congestion and edema of the left nipple-areola complex four hours after undergoing bilateral reduction mammaplasty. Examination shows rapid capillary refill. Which of the following surgical interventions is the most appropriate initial management?

(A) Debridement and healing by second intention
(B) Release of the suture line and exploration of the pedicle
(C) Conversion of the nipple-areola complex to a free composite graft
(D) Central wedge resection under the nipple-areola complex
(E) Removal of additional tissue from the breast

A

The correct response is Option B.

Loss of the nipple-areola complex after reduction mammaplasty may be caused by torsion of the pedicle or by excessive tension on the closure. In the first hours after reduction mammaplasty, attempted exploration of the pedicle should be performed to release any possible tension. A blue and engorged nipple may be the first sign of impending necrosis. Other methods such as use of nitroglycerin patch may aid vasodilatation of the vessels.

Conversion of the pedicle/nipple-areola complex to a free composite graft should be considered if release of the pedicle does not yield any improvement in the blood flow or if no correctable cause can be identified. Debridement and open packing of the wound is an appropriate management option when the tissues are already necrotic. Further reduction mammaplasty would release the tension on the skin flaps but would not necessarily improve the vascularity of the nipple-areola complex. Wedge resection of the central portion of the breast usually leaves unsightly scarring superior to the nipple-areola complex.

59
Q

A 15-year-old girl comes to the office with a six-month history of sudden, rapid, asymmetric enlargement of her left breast. On physical examination, there is a large palpable mass occupying the left breast, which also has marked nipple-areola stretching, prominent dilated veins, and skin ulceration superolateral to the nipple. Mammograms and sonograms show a dense, circumscribed, 6-cm-diameter homogeneous mass occupying the left breast. Which of the following is the most likely diagnosis?
(A) Carcinoma
(B) Cyst
(C) Giant fibroadenoma
(D) Juvenile breast hypertrophy
(E) Phyllodes tumor

A

The correct response is Option C.

The differential diagnosis of a large lesion in the breast of an adolescent girl includes giant fibroadenoma, phyllodes tumor, and virginal hypertrophy. Fibroadenoma is the most common breast neoplasm in the adolescent patient, and giant fibroadenoma is characterized by size greater than 5.0 cm in diameter, presentation at or soon after puberty, and short doubling time. The lesion is usually solitary, firm, and nontender and presents as a rapid asymmetric breast enlargement with prominent veins over the tumor and occasional skin ulceration due to pressure. Giant fibroadenomas are benign lesions that can be excised by enucleation with minimal risk of local recurrence. Mastectomy is not necessary for management of these lesions, and no adjuvant treatment is indicated.

Phyllodes tumors are large, benign tumors that occur primarily in the perimenopausal patient. They are histologically distinct from giant fibroadenomas, and atypical changes in these tumors are rare in the adolescent.

Juvenile breast hypertrophy is a rare but well-described entity in young, early pubertal girls. It presents as diffuse enlargement of the breast without any nodularity or presence of a discrete mass. Management is reduction mammaplasty.

60
Q

A 25-year-old woman comes to the office because she has loss of sensation in the nipple-areola complexes three months after she underwent bilateral reduction mammaplasty. Which of the following intercostal nerves were most likely injured during the procedure?
(A) First and second
(B) Third and fourth
(C) Fifth and sixth
(D) Seventh and eighth
(E) Ninth and tenth

A

The correct response is Option B.

Recently, two separate groups have shown patterns of cutaneous innervation of the breast through detailed anatomic studies in cadavers. In general, cutaneous branches of the intercostal nerves are noted to pass through the deep fascia of the chest wall at two anatomic points: the lateral cutaneous branches at the midaxillary line and the anterior cutaneous branches beside the sternum. The breast skin is innervated by the lateral and anterior cutaneous branches of the T1 through T7 intercostal nerves. However, the contributions of the T1 and T7 intercostal nerves are small, and thus the branches of the T2 through T6 intercostal nerves are likely more important. There is no contribution by the T8 through T12 intercostal nerves to innervation of the breast skin.

Innervation to the nipple-areola complex is supplied by the anterior and lateral cutaneous branches of the T3 through T5 intercostal nerves. While all of these nerves supply branches, the T4 intercostal nerve is the most consistent in its contribution to innervation to the nipple-areola complex. Also, the cutaneous area of sensation of the nipple-areola complex, supplied by the T4 intercostal nerve branches, is larger than that supplied by the T3 or T5 intercostal nerve branches. Therefore, the T4 intercostal nerve is the primary nerve for cutaneous innervation to the nipple-areola complex, but it is not the sole supplier of this innervation.

61
Q

On the basis of evaluation of cutaneous light-pressure thresholds of the breast with Semmes-Weinstein monofilaments, which of the following areas of the breast is most sensitive?

(A) Areola
(B) Inferior quadrants
(C) Nipple
(D) Superior quadrants

A

The correct response is Option D.

Based on the evaluation of cutaneous light-pressure thresholds with the Semmes-Weinstein monofilaments, there are marked differences in sensory perception between the skin of the breast, the areola, and the nipple. Irrespective of breast size, the skin in the superior quadrant is the most sensitive part of the breast, the areola is less sensitive, and the nipple is the least sensitive part to light pressure. The fact that the nipple is the least sensitive area in the female breast is thought to allow the mother to nurse without discomfort. Vibration is most sensitive in the areola.
Larger breasts are significantly less sensitive than smaller breasts in all anatomic areas, and there is a significant decrease of sensibility with increasing breast ptosis. Sensitivity tends to decrease with age.

62
Q

A 15-year-old girl has a one-year history of asymmetric enlargement of the left breast. Physical examination shows a 10-cm mass as well as ptosis, marked stretching of the nipple-areola complex, skin ulceration superolateral to the nipple, and presence of a prominent, dilated vein. Mammography and ultrasonography show a dense, circumscribed, homogeneous mass occupying the entire breast. Which of the following is the most appropriate management?

(A) Enucleation of the mass
(B) Hormone therapy
(C) Lumpectomy and postoperative radiation therapy
(D) Reduction mammaplasty
(E) Total mastectomy and reconstruction

A

The correct response is Option A.

This patient has giant fibroadenoma, which is best managed by enucleation of the mass. A large breast lesion in a female adolescent may result from giant fibroadenoma, phyllodes tumor, and juvenile breast hypertrophy. Fibroadenoma is the most common breast neoplasm in adolescents; giant fibroadenoma is characterized by a lesion larger than 5 cm in diameter, presentation at or soon after puberty, and a short doubling time. Usually, the lesion is solitary, firm, and nontender and causes rapid, asymmetric enlargement of the breast, prominent overlying veins, and occasionally skin ulceration caused by pressure. Giant fibroadenomas are benign lesions that can be excised by enucleation with minimal risk of recurrence. They do not require mammaplasty, lumpectomy, mastectomy, or hormone or radiation therapy.

Phyllodes tumors are large, benign tumors that occur primarily in perimenopausal patients. They are histologically distinct from giant fibroadenomas and rarely affect adolescents. They are treated with local excision or mastectomy.

Juvenile breast hypertrophy is a rare but well-described disorder in young girls early in puberty. It presents as diffuse enlargement of the breast without nodules or discrete masses. It is treated by reduction mammaplasty.

63
Q

A 20-year-old woman who comes to the office for consultation regarding reduction mammaplasty inquires about ability to breast-feed after surgery. The most appropriate response to this patient is that breast-feeding is possible after each of the following techniques EXCEPT

(A) free nipple
(B) inferior pedicle
(C) liposuction
(D) McKissock bipedicle
(E) vertical

A

The correct response is Option A.

In the reduction mammaplasty technique using free nipple grafting, the nipple is totally removed from the breast and placed as a skin graft in a new, superior location. Therefore, lactation is no longer possible because the milk ducts are no longer connected to the nipple.

If suction lipectomy or pedicled techniques are used, breast tissue is preserved behind the nipple and the collecting ducts are still attached to the nipple; therefore, lactation is possible. Even with these techniques, the success of breast-feeding still varies with societal influences, patient desires, amount of tissue resected, and hormonal production of engorgement and milk.

In one study of 78 patients having babies after reduction, 29% breast-fed their children, 18% attempted breast-feeding and were unsuccessful, and 52% did not wish to breast-feed. In a study of 49 Brazilian women who had undergone reduction mammaplasty (breast-feeding is more common in Brazil), 58% breast-fed compared with 94% of nonoperated control subjects. The length of time that children were breast-fed was shorter in the reduction mammaplasty patients. In a third study of 30 women, 93% wished to breast-feed and were successful, although many of the babies required complementary formula feedings.

64
Q

In patients who have undergone bilateral reduction mammaplasty using an inferior pedicle technique, which of the following findings is most likely to be identified on mammography six to 18 months after surgery?

(A) Calcifications
(B) Fat necrosis
(C) Oil cysts
(D) Periareolar fibrosis
(E) Skin thickening

A

The correct response is Option A.

Mammography is recommended in women older than age 35 years for detection of carcinoma. One recent study, involving women who underwent bilateral reduction mammaplasty using an inferior pedicle technique, examined findings on radiographs of the breasts six to 18 months after surgery. According to the results of this study, the most common findings were parenchymal redistribution (90.2%) and elevation of the nipple (84.9%) caused by a downward shifting of the breast tissue. Calcifications occurred in 25.6% of patients, retroareolar fibrotic banding from the
transposed flap in 20.3% of patients, and oil cysts resulting from localized fat necrosis in 19.4% of patients. Thickening of the skin and areola caused by scar tissue were less common findings. Because such studies have delineated the “routine” mammography findings in patients who have undergone reduction mammaplasty, lesions can be diagnosed more easily and unnecessary biopsy procedures can be avoided.

65
Q

A 13-year-old boy with gynecomastia desires corrective surgery. An initial preoperative evaluation of this patient should include examination of which of the following?

(A) Eyes
(B) Thyroid gland
(C) Pectoralis muscle
(D) Abdomen
(E) Genitalia

A

The correct response is Option E.

The genitalia should be examined in this 13-year-old boy with gynecomastia. Although most adolescent boys with breast development do not exhibit other signs of feminization caused by hormonal excess or other genetic problems, examination of the genitalia could reveal an underlying cause of the gynecomastia. Testicular tumors are typically palpable on physical examination, and some adolescent boys will have nonpalpable and/or undescended testes. In patients with these findings, genetic and/or endocrine evaluation is required before any operative procedures are performed.

Visual fields may be altered in a patient who has a pituitary tumor, and patients with adrenal tumors may have palpable abdominal masses. Examination of these sites should be performed secondarily to examination of the genitalia.

Thyroid abnormalities are not associated with gynecomastia. The pectoralis muscle may be partially absent in patients with Poland syndrome but not in patients with gynecomastia.

66
Q

A 35-year-old woman is scheduled to undergo reduction mammaplasty with removal of approximately 700 g of tissue bilaterally using an inferior pedicle technique. During preoperative marking of the patient, which of the following points should be used as a landmark to best determine the new position of the nipple?

(A) 7 cm above the inframammary fold
(B) 21 cm below the sternal notch
(C) Inframammary fold
(D) Midhumerus
(E) Xiphoid

A

The correct response is Option C.

In patients undergoing reduction mammaplasty, the new nipple position should be determined when the patient is in an upright position, before the procedure is performed. During preoperative marking, the inframammary fold is used as a landmark to determine the most appropriate position of the nipple on the midbreast line. This is consistent regardless of the reduction technique performed.

The midhumerus, the xiphoid, and the measured distance below the sternal notch (19 to 23 cm) are helpful but less reliable landmarks. The measurement of 7 cm above the inframammary fold is essential during surgery, when the new nipple position is being marked during closure of the incisions, but is not a reliable preoperative measurement.

67
Q

The sensation to the nipple-areolar complex is derived from the

(A) anterior cutaneous branches of the sixth and seventh intercostal nerves
(B) anterolateral branches of the third through fifth intercostal nerves
(C) branches of the lateral pectoral nerve
(D) nerves of the cervical plexus
(E) nerves traveling with the internal mammary artery

A

The correct response is Option B.

The sensation to the nipple-areolar complex is derived primarily from the fourth intercostal nerve, with lesser contributions from the third and fifth intercostal nerves. The superior portion of the breast derives its sensation from the nerves of the cervical plexus; sensory innervation to the medial skin and inframammary gland is provided by the anterior cutaneous branches of the second through seventh intercostal nerves.

68
Q

A 16-year-old girl has had rapid, asymmetric enlargement of the left breast over the past year. On physical examination, an 18-cm mass can be palpated; there is ptosis and stretching of the nipple-areola complex. The veins are prominent and dilated, and there is ulceration of the skin superolateral to the nipple. Mammography shows a dense, circumscribed, homogeneous mass that encompasses the entire breast.

Which of the following is the most appropriate management?

(A) Hormone therapy
(B) Enucleation
(C) Subcutaneous mastectomy
(D) Simple mastectomy
(E) Reduction mammaplasty

A

The correct response is Option B.

This 16-year-old girl has fibroadenoma, which is the most common neoplasm of the breast in adolescents. Giant fibroadenomas are typically solitary, firm, nontender, benign lesions that develop at or soon after the onset of puberty. They are larger than 5 cm in diameter and double in size within a short time. Rapid enlargement of one breast is characteristic. Prominent veins are noted over the arc of the tumor; some patients develop skin ulcerations because of the pressure caused by the fibroadenoma. Enucleation is curative, and the risk for local recurrence is minimal.
Hormone therapy would only stimulate growth of the glands within the breast.

Mastectomy is excessive and unnecessary in patients with giant fibroadenoma.

Reduction mammaplasty is indicated for management of juvenile breast hypertrophy, which is characterized by diffuse enlargement of the breast without a palpable mass or nodes.

69
Q

Sensation to the nipple-areola complex is provided primarily by which of the following nerves?

(A) Anterior cutaneous nerve from T3
(B) Anterior cutaneous nerve from T4
(C) Lateral cutaneous nerve from T4
(D) Medial cutaneous nerve from T5
(E) Posterior cutaneous nerve from T5

A

The correct response is Option C.

Sensation to the nipple-areola complex is primarily provided by the lateral cutaneous nerve from T4. The innervation of the skin of the breast is segmental and is derived from the dermatomes associated with breast development. Knowledge of this innervation is crucial before performing breast surgery.

The upper breast receives its sensation from the supraclavicular nerves that originate from the third and fourth branches of the cervical plexus. The medial and inferior aspects of the breast are innervated by small medial branches of the anterior cutaneous nerves. The lateral cutaneous branches course subcutaneously to provide sensation to the midclavicular region and contribute to the innervation of the areola.

70
Q

A 40-year-old woman has cyanosis of the right nipple one hour after undergoing bilateral breast reduction with removal of 1500 g of tissue on each side. Which of the following is the most appropriate management?

(A) Observation
(B) Application of leeches
(C) Hyperbaric oxygen therapy
(D) Release of the sutures
(E) Conversion of the nipple-areola complex to a split-thickness skin graft

A

The correct response is Option D.

The findings seen in this patient are consistent with necrosis of the nipple-areola complex. This condition can be caused by direct devascularization of the breast mound, torsion, or direct pressure on the nipple and areola. Appropriate management includes immediate decompression with release of the sutures and operative exploration. Excision of the nipple-areola complex with subsequent defatting and conversion to a full-thickness skin graft is indicated for those patients who have ischemia of the breast mound identified on operative exploration.

Free nipple grafting may be considered to decrease the risk for necrosis of the nipple-areola complex in women undergoing breast reduction with an estimated volume of resection of more than 1500 g on each side, a nipple transposition length of 25 cm or greater, or associated risk factors such as smoking or diabetes mellitus.

Although leeches can be used to relieve venous insufficiency, this process would need to be undertaken over several days to be effective. Hyperbaric oxygen therapy increases the oxygen content of the tissue but is also a lengthy process.

71
Q

Gynecomastia is an adverse effect of administration of each of the following agents EXCEPT

(A) cimetidine (Tagamet)
(B) digitalis (Digoxin)
(C) minocycline (Minocin)
(D) spironolactone (Aldactone)
(E) zolpidem (Ambien)

A

The correct response is Option E.

Many agents have been linked to gynecomastia, such as amphetamines, cimetidine, digitalis, haloperidol, isoniazid, methyldopa, opiates, progestins, spironolactone, and tricyclic antidepressants. Associated conditions include obesity, liver disease, kidney failure, adrenal tumors, hyperthyroidism, and hypothyroidism. Gynecomastia can also be caused by increased estrogen levels (men with testicular tumors or who use androgen-based agents) or decreased estrogen levels (men with Klinefelter syndrome or who undergo orchiectomy).

Zolpidem has not been shown to be a cause of gynecomastia.

72
Q

Which of the following is associated with reduction mammaplasty using the vertical scar (Lejour) technique?

(A) Central vertical glandular excision
(B) Inferiorly based blood supply to the nipple
(C) Keyhole-pattern skin excision
(D) Precision in determining the endpoint of resection
(E) Wide periareolar skin excision

A

The correct response is Option A.

Features of the vertical (Lejour) mammaplasty include central vertical glandular excision to improve postoperative shape (by narrowing the breast while maximizing breast projection) and excision of skin in one direction only to decrease scar burden. Vertical mammaplasty is a technique of central breast reduction with undermining of the lower skin, as well as use of adjustable markings and an upper pedicle to maintain the blood supply to the areola.

Keyhole-pattern and wide periareolar resections are not features of the vertical mammaplasty; therefore, circumareolar scar quality is not compromised by excess skin tension. However, because of the central and posterior resection used with this technique, it is more difficult to determine the endpoint of resection.

73
Q

A 58-year-old man has had moderate gynecomastia with severe skin redundancy for the past eight years. A photograph is shown above. Complete physical examination and laboratory studies show no other abnormalities. Which of the following is the LEAST acceptable technique for management?

(A) Suction lipectomy with subsequent skin shrinkage
(B) Concentric circle resection
(C) Wise-pattern mastopexy
(D) Glandular resection through an areolar incision with adjunctive suction lipectomy
(E) Breast amputation and free nipple grafting

A

The correct response is Option C.

Gynecomastia can be classified according to three grades. Grade I gynecomastia involves visible mild breast enlargement without skin redundancy. In grade IIA gynecomastia, there is moderate breast enlargement without skin redundancy; in grade IIB gynecomastia, there is moderate breast enlargement with skin redundancy. Grade III gynecomastia is characterized by marked breast enlargement with marked skin redundancy. Although most
adolescents with gynecomastia have regression within two years (only 7.7% of affected adolescents have duration of symptoms for a longer time), regression is unlikely to be seen in this older patient, who has had severe ptosis for the past eight years.

Suction lipectomy has eliminated the need for skin resection in many gynecomastia patients, especially adolescents. Fibrous enlargement can be managed with glandular resection through an areolar incision with adjunctive suction lipectomy. However, skin resection is still recommended in older patients with grade III gynecomastia who have significant ptosis. Other procedures, such as resection of a concentric circle of skin, pedicled relocation of the nipple with skin resection, or breast amputation with free nipple grafting, may be considered. The Wise-pattern mastopexy is used to create a projecting, conical breast in women undergoing breast reduction and should not be performed in gynecomastia patients who require a breast elimination procedure.

74
Q

Which of the following long-term complications results in the greatest patient dissatisfaction following reduction mammaplasty?

(A) “Bottoming out” of the breasts
(B) Excessive reduction
(C) Inadequate reduction
(D) Loss of nipple sensation
(E) Prominent scarring

A

The correct response is Option E.

According to several retrospective studies, prominent scarring is the most common source of patient dissatisfaction following reduction mammaplasty. “Bottoming out” of the breasts, excessive or inadequate reduction, and loss of sensation are less commonly reported as sources of patient dissatisfaction. Although patient satisfaction is generally high on follow-up evaluation, preoperative discussion of the potential long-term complications of reduction mammaplasty should be a priority for the surgeon. Long-term complications such as those mentioned above have been shown to significantly affect the patient’s perception of the postoperative results.

75
Q

A 30-year-old woman who underwent reduction mammaplasty using the inferior pedicle technique 10 years ago is pregnant with her first child. The probability that she will breast-feed is closest to

(A) 10%
(B) 30%
(C) 50%
(D) 70%
(E) 90%

A

The correct response is Option B.

A large proportion of the women who seek breast reduction surgery are of child-bearing age. Because of the known advantages of breast-feeding, much attention has been directed toward the possibility of breast-feeding after reduction mammaplasty. In North American studies, 27% to 35% of postreduction patients were reported to have breast-fed in excess of two months. The most recent data indicate that 29% of women who had undergone inferior pedicle reduction mammaplasty (the procedure used on this patient) breast-fed either exclusively or with formula supplementation, 18% were unsuccessful in their attempt to breast-feed, and 53% did not attempt breast-feeding.

Preservation of adequate breast tissue connected to the nipple was thought to be paramount to subsequent breast-feeding success. There was no significant difference in the mass or volume of tissue excised between the groups that breast-fed and those with an unsuccessful breast-feeding attempt. No decision about breast-feeding was attributable to perioperative complications such as wound infection or necrosis. The prevalence of breast-feeding among postreduction mammaplasty patients falls near the breast-feeding rate found in the population of child-bearing women who did not undergo breast surgery.

76
Q
A